LSAT and Law School Admissions Forum

Get expert LSAT preparation and law school admissions advice from PowerScore Test Preparation.

User avatar
 Dave Killoran
PowerScore Staff
  • PowerScore Staff
  • Posts: 5853
  • Joined: Mar 25, 2011
|
#45674
Complete Question Explanation
(The complete setup for this game can be found here: lsat/viewtopic.php?t=2357)

The correct answer choice is (A)

If the first reassignment is from Plan 1, then the second reassignment follows Plan 2 or Plan 3. Respectively, these give:

..... ..... ..... ..... After Plan 1 then Plan 2: ..... Q: N ..... R: O ..... S: M ..... T: L

..... ..... ..... ..... After Plan 1 then Plan 3: ..... Q: N ..... R: O ..... S: M ..... T: L


Answer choice (A) is correct because in both Plan 2 and Plan 3 (following Plan 1) L is assigned to T.

Get the most out of your LSAT Prep Plus subscription.

Analyze and track your performance with our Testing and Analytics Package.